Solving $472x ≡ 32 ;(text{mod } 92)$: How to solve when $a$ is greater than $m$












2












$begingroup$


I'm not sure where to begin here. I think that I can change the $472x$ to be $12x$ because they should be equal in mod $92$. Then, I believe I can simplify the equation to be: $3x ≡ 8 ; (text{mod } 23)$










share|cite|improve this question











$endgroup$












  • $begingroup$
    You're almost done: $bmod 23!:, 3xequiv 8equiv -15,$ so $,xequiv -5 $
    $endgroup$
    – Bill Dubuque
    Dec 26 '18 at 0:47


















2












$begingroup$


I'm not sure where to begin here. I think that I can change the $472x$ to be $12x$ because they should be equal in mod $92$. Then, I believe I can simplify the equation to be: $3x ≡ 8 ; (text{mod } 23)$










share|cite|improve this question











$endgroup$












  • $begingroup$
    You're almost done: $bmod 23!:, 3xequiv 8equiv -15,$ so $,xequiv -5 $
    $endgroup$
    – Bill Dubuque
    Dec 26 '18 at 0:47
















2












2








2





$begingroup$


I'm not sure where to begin here. I think that I can change the $472x$ to be $12x$ because they should be equal in mod $92$. Then, I believe I can simplify the equation to be: $3x ≡ 8 ; (text{mod } 23)$










share|cite|improve this question











$endgroup$




I'm not sure where to begin here. I think that I can change the $472x$ to be $12x$ because they should be equal in mod $92$. Then, I believe I can simplify the equation to be: $3x ≡ 8 ; (text{mod } 23)$







modular-arithmetic






share|cite|improve this question















share|cite|improve this question













share|cite|improve this question




share|cite|improve this question








edited Dec 10 '18 at 20:56









MisterRiemann

5,8121625




5,8121625










asked Dec 10 '18 at 20:39









user52640user52640

453




453












  • $begingroup$
    You're almost done: $bmod 23!:, 3xequiv 8equiv -15,$ so $,xequiv -5 $
    $endgroup$
    – Bill Dubuque
    Dec 26 '18 at 0:47




















  • $begingroup$
    You're almost done: $bmod 23!:, 3xequiv 8equiv -15,$ so $,xequiv -5 $
    $endgroup$
    – Bill Dubuque
    Dec 26 '18 at 0:47


















$begingroup$
You're almost done: $bmod 23!:, 3xequiv 8equiv -15,$ so $,xequiv -5 $
$endgroup$
– Bill Dubuque
Dec 26 '18 at 0:47






$begingroup$
You're almost done: $bmod 23!:, 3xequiv 8equiv -15,$ so $,xequiv -5 $
$endgroup$
– Bill Dubuque
Dec 26 '18 at 0:47












1 Answer
1






active

oldest

votes


















0












$begingroup$

You're on the right track. Starting from
$$ 3x equiv 8 ; (text{mod } 23), $$
you should find the multiplicative inverse $3^{-1}$ of $3$ modulo $23$, i.e. the number satisfying
$$ 3^{-1} cdot 3 equiv 1 ; (text{mod } 23). $$
This can be done with the help of the Euclidean algorithm, or alternatively you could just notice that
$$ 8cdot 3 = 24 equiv 1 ; (text{mod } 23), $$
so $3^{-1} = 8$ in this case. Then you can multiply both sides of your equation by this multiplicative inverse to obtain
$$ x equiv 8cdot 3 x equiv 8 cdot 8 = 64 equiv 18 ; (text{mod } 23). $$
Hence $x = 18+23k$, for an integer $k$, so you get four incongruent solutions (modulo $92$) to your original equation, namely by inserting $k=0,1,2,3$.






share|cite|improve this answer









$endgroup$













    Your Answer





    StackExchange.ifUsing("editor", function () {
    return StackExchange.using("mathjaxEditing", function () {
    StackExchange.MarkdownEditor.creationCallbacks.add(function (editor, postfix) {
    StackExchange.mathjaxEditing.prepareWmdForMathJax(editor, postfix, [["$", "$"], ["\\(","\\)"]]);
    });
    });
    }, "mathjax-editing");

    StackExchange.ready(function() {
    var channelOptions = {
    tags: "".split(" "),
    id: "69"
    };
    initTagRenderer("".split(" "), "".split(" "), channelOptions);

    StackExchange.using("externalEditor", function() {
    // Have to fire editor after snippets, if snippets enabled
    if (StackExchange.settings.snippets.snippetsEnabled) {
    StackExchange.using("snippets", function() {
    createEditor();
    });
    }
    else {
    createEditor();
    }
    });

    function createEditor() {
    StackExchange.prepareEditor({
    heartbeatType: 'answer',
    autoActivateHeartbeat: false,
    convertImagesToLinks: true,
    noModals: true,
    showLowRepImageUploadWarning: true,
    reputationToPostImages: 10,
    bindNavPrevention: true,
    postfix: "",
    imageUploader: {
    brandingHtml: "Powered by u003ca class="icon-imgur-white" href="https://imgur.com/"u003eu003c/au003e",
    contentPolicyHtml: "User contributions licensed under u003ca href="https://creativecommons.org/licenses/by-sa/3.0/"u003ecc by-sa 3.0 with attribution requiredu003c/au003e u003ca href="https://stackoverflow.com/legal/content-policy"u003e(content policy)u003c/au003e",
    allowUrls: true
    },
    noCode: true, onDemand: true,
    discardSelector: ".discard-answer"
    ,immediatelyShowMarkdownHelp:true
    });


    }
    });














    draft saved

    draft discarded


















    StackExchange.ready(
    function () {
    StackExchange.openid.initPostLogin('.new-post-login', 'https%3a%2f%2fmath.stackexchange.com%2fquestions%2f3034431%2fsolving-472x-%25e2%2589%25a1-32-textmod-92-how-to-solve-when-a-is-greater-than-m%23new-answer', 'question_page');
    }
    );

    Post as a guest















    Required, but never shown

























    1 Answer
    1






    active

    oldest

    votes








    1 Answer
    1






    active

    oldest

    votes









    active

    oldest

    votes






    active

    oldest

    votes









    0












    $begingroup$

    You're on the right track. Starting from
    $$ 3x equiv 8 ; (text{mod } 23), $$
    you should find the multiplicative inverse $3^{-1}$ of $3$ modulo $23$, i.e. the number satisfying
    $$ 3^{-1} cdot 3 equiv 1 ; (text{mod } 23). $$
    This can be done with the help of the Euclidean algorithm, or alternatively you could just notice that
    $$ 8cdot 3 = 24 equiv 1 ; (text{mod } 23), $$
    so $3^{-1} = 8$ in this case. Then you can multiply both sides of your equation by this multiplicative inverse to obtain
    $$ x equiv 8cdot 3 x equiv 8 cdot 8 = 64 equiv 18 ; (text{mod } 23). $$
    Hence $x = 18+23k$, for an integer $k$, so you get four incongruent solutions (modulo $92$) to your original equation, namely by inserting $k=0,1,2,3$.






    share|cite|improve this answer









    $endgroup$


















      0












      $begingroup$

      You're on the right track. Starting from
      $$ 3x equiv 8 ; (text{mod } 23), $$
      you should find the multiplicative inverse $3^{-1}$ of $3$ modulo $23$, i.e. the number satisfying
      $$ 3^{-1} cdot 3 equiv 1 ; (text{mod } 23). $$
      This can be done with the help of the Euclidean algorithm, or alternatively you could just notice that
      $$ 8cdot 3 = 24 equiv 1 ; (text{mod } 23), $$
      so $3^{-1} = 8$ in this case. Then you can multiply both sides of your equation by this multiplicative inverse to obtain
      $$ x equiv 8cdot 3 x equiv 8 cdot 8 = 64 equiv 18 ; (text{mod } 23). $$
      Hence $x = 18+23k$, for an integer $k$, so you get four incongruent solutions (modulo $92$) to your original equation, namely by inserting $k=0,1,2,3$.






      share|cite|improve this answer









      $endgroup$
















        0












        0








        0





        $begingroup$

        You're on the right track. Starting from
        $$ 3x equiv 8 ; (text{mod } 23), $$
        you should find the multiplicative inverse $3^{-1}$ of $3$ modulo $23$, i.e. the number satisfying
        $$ 3^{-1} cdot 3 equiv 1 ; (text{mod } 23). $$
        This can be done with the help of the Euclidean algorithm, or alternatively you could just notice that
        $$ 8cdot 3 = 24 equiv 1 ; (text{mod } 23), $$
        so $3^{-1} = 8$ in this case. Then you can multiply both sides of your equation by this multiplicative inverse to obtain
        $$ x equiv 8cdot 3 x equiv 8 cdot 8 = 64 equiv 18 ; (text{mod } 23). $$
        Hence $x = 18+23k$, for an integer $k$, so you get four incongruent solutions (modulo $92$) to your original equation, namely by inserting $k=0,1,2,3$.






        share|cite|improve this answer









        $endgroup$



        You're on the right track. Starting from
        $$ 3x equiv 8 ; (text{mod } 23), $$
        you should find the multiplicative inverse $3^{-1}$ of $3$ modulo $23$, i.e. the number satisfying
        $$ 3^{-1} cdot 3 equiv 1 ; (text{mod } 23). $$
        This can be done with the help of the Euclidean algorithm, or alternatively you could just notice that
        $$ 8cdot 3 = 24 equiv 1 ; (text{mod } 23), $$
        so $3^{-1} = 8$ in this case. Then you can multiply both sides of your equation by this multiplicative inverse to obtain
        $$ x equiv 8cdot 3 x equiv 8 cdot 8 = 64 equiv 18 ; (text{mod } 23). $$
        Hence $x = 18+23k$, for an integer $k$, so you get four incongruent solutions (modulo $92$) to your original equation, namely by inserting $k=0,1,2,3$.







        share|cite|improve this answer












        share|cite|improve this answer



        share|cite|improve this answer










        answered Dec 10 '18 at 20:46









        MisterRiemannMisterRiemann

        5,8121625




        5,8121625






























            draft saved

            draft discarded




















































            Thanks for contributing an answer to Mathematics Stack Exchange!


            • Please be sure to answer the question. Provide details and share your research!

            But avoid



            • Asking for help, clarification, or responding to other answers.

            • Making statements based on opinion; back them up with references or personal experience.


            Use MathJax to format equations. MathJax reference.


            To learn more, see our tips on writing great answers.




            draft saved


            draft discarded














            StackExchange.ready(
            function () {
            StackExchange.openid.initPostLogin('.new-post-login', 'https%3a%2f%2fmath.stackexchange.com%2fquestions%2f3034431%2fsolving-472x-%25e2%2589%25a1-32-textmod-92-how-to-solve-when-a-is-greater-than-m%23new-answer', 'question_page');
            }
            );

            Post as a guest















            Required, but never shown





















































            Required, but never shown














            Required, but never shown












            Required, but never shown







            Required, but never shown

































            Required, but never shown














            Required, but never shown












            Required, but never shown







            Required, but never shown







            Popular posts from this blog

            Le Mesnil-Réaume

            Ida-Boy-Ed-Garten

            web3.py web3.isConnected() returns false always